what is the total capacity in gallows of a rectangular tank 5 meters in height knowing that it will hold 200 liters per foot?

Answers

Answer 1

729.1  is the total capacity in gallows of a rectangular tank .

What is rectangle?

The internal angles of a rectangle, which has four sides, are all exactly 90 degrees. At each corner or vertex, the two sides come together at a straight angle. The rectangle differs from a square because its two opposite sides are of equal length.

1 Foot height  can store  =  200 ltrs

Height of tank = 5 m = 500/30.48 = 16.4 ft

Total Capacity = 200 * 16.4 =3280.8 ltr

There were  several gallons in use. One gallons  is roughly =  4.5 ltr

Hence the  capacity in  gallon  = 3280.8/4.5 = 729.1

Learn more about rectangle

brainly.com/question/15019502

#SPJ9


Related Questions

Drag statements and reasons to each row to show why the slope of the line between R and S is the same as the slope between S and T, given that triangles A and B are similar.

Answers

For statement 5/3, the reason is the definition of slope. And for statement (5/3)=(15/9), the reason is triangle A is similar to triangle B.

What is the slope?

Slope or the gradient is the number or the ratio which determines the direction or the steepness of the line. The slope is also defined as the ratio of the rise to the run.

If the slope of the triangles is equal then triangle A is similar to triangle B.

In the statement  when we simplify

(5/3)=(15/9),

(5/3)=(5/3),

Then the reason for this statement is "If the slope of triangles are equal"

And the definition of slope in general is m = y/x.

Therefore, for statement 5/3, the reason is the definition of slope. And for statement (5/3)=(15/9), the reason is triangle A is similar to triangle B.

To know more about slope follow

https://brainly.com/question/3493733

#SPJ1

Drag statements and reasons to each row to show why the slope of the line between R and S is the same

Are investment funds a liability or asset?

Asset - Things you can sell for value
Liabitlies - You have to pay no matter what

Answers

Answer:

Asset

Step-by-step explanation:

Because your selling your money to in something to make it have value.

How can I solve this problem?

How can I solve this problem?

Answers

The answer should be 137/36

A particular fruit's weights are normally distributed, with a mean of 615 grams and a standard deviation of 10 grams.

The heaviest 10% of fruits weigh more than how many grams?

Give your answer to the nearest 0.1 gram.

Answers

The heaviest 10% of fruits weigh more than 814.79 grams.

The problem setup implies that the weight of this fruit is a random variable when it says a particular fruit's weights are normally distributed.

What are the formula weights are normally distributed?

Define X = Random Variable of the fruit's weight

\(X \sim N(mean = 760, standard deviation = 39)\)

We want to find the specific gram value for which 8% of the fruits are heavier

The 92nd percentile of X's normal distribution

\(Solve for x: Pr( X < x ) = Normal( 760, 39 ) CDF = .92\)

Standardizing we obtain

\(Pr( [X - 760]/39 < [x - 760]/39 ) = Pr( Z < [x - 760]/39 ) = .92,\)

where Z is a standard normal random variable

\(Pr( Z < [x - 760]/39 ) = Standard \ Normal \ CDF( [x - 760]/39 )\)

==> Set this equal to .92:

\(Standard \ Normal \ CDF( [x - 760]/39 ) = .92\)

Then, from the standard normal table:

\([x - 760]/39 = Inverse Standard Normal\)

\(CDF( .92 ) = 1.405\)

Use algebra to solve for x:

\(x = 39 * 1.405 + 760 = 814.795\)

Therefore the heaviest 10% of fruits weigh more than 814.79 grams.

To learn more about the weights normally distributed visit:

https://brainly.com/question/17091450

#SPJ1

the scatterplot below shows the number of hours that students read weekly and the number of hours that they spend on chores weekly​

Answers

Answer:

C. In general, the number of hours of reading does not affect the number of hours used for chores.

Step-by-step explanation:

I just did this (edge) and got it right.   hope this helps:)

Answer:

C. In general, the number of hours of reading does not affect the number of hours used for chores.

Step-by-step explanation:

sorry if im wrong

Can't seem to understand this, please help!

Can't seem to understand this, please help!

Answers

The equations of each line are:

a. y = -3x + 3

b. y = -2/3x - 3

How to Write the Equation of a Line?

In slope-intercept form, the equation of a line is written as, y = mx + b, given that:

m is the slope valueb is the y-intercept value

Note: the slope values of parallel lines are the same.

a. The slope of y = -3x - 1 is, -3. The line that is parallel to it will have the same slope (m) of -3.

Substitute m = -3 and (x, y) = (2, -3) into y = mx + b to find b:

-3 = -3(2) + b

-3 = -6 + b

-3 + 6 = b

3 = b

b = 3

Substitute b = 3 and m = -3 into y = mx + b

y = -3x + 3

b. Rewrite 2x + 3y = 6 in the slope-intercept form

3y = -2x + 6

y = -2/3y + 2

The slope of y = -2/3y + 2 is, -2/3. The line that is parallel to it will have the same slope (m) of -2/3.

Substitute m = -2/3 and (x, y) = (0, 5) into y = mx + b to find b:

5 = -2/3(0) + b

-3 = 0 + b

-3 = b

b = -3

Substitute b = -3 and m = -2/3 into y = mx + b

y = -2/3x - 3

Learn more about the equation of a line on:

https://brainly.com/question/13763238

#SPJ1

If x = 2, y = 3 and z = -5, find the value of square root of x + y squared + z squared

Answers

The value of square root of x + y squared + z squared is 30

How to solve algebra?

x = 2, y = 3 and z = -5

\(( \sqrt{x + y} )^{2} + z ^{2} \)

substitute the value of x, y and z

\( = ( \sqrt{2 + 3} )^{2} + - 5 ^{2} \)

simplify the square root and square

\( = (2 + 3) + 25\)

\( = 5 + 25\)

\( = 30\)

Ultimately, x + y squared + z squared is 30

Read more on algebra:

https://brainly.com/question/4344214

#SPJ1

Ramesh dan syarika ans. Ramesh takes a medical and health insurance policy with a deductible of RM1 500 per year and co-insurance of 20% of covered medical expenses. Ramesh went to the hospital for the first time for knee treatment and the medical cost was RM700, the medical cost for the second and third treatments was RM2 000 and RM1 700 respectively. The three treatments were in the same year. How much medical expenses should be borne by Ramesh and the insurance company? ​

Answers

Ramesh will bear a total medical expense of RM1,760, and the insurance company will cover RM2,900.

1. Ramesh's deductible is RM1,500 per year. This means that he is responsible for paying the first RM1,500 of medical expenses before the insurance coverage kicks in.

2. For the first treatment, the medical cost was RM700. Since this amount is less than the deductible, Ramesh will have to pay the entire RM700 out of pocket.

3. For the second treatment, the medical cost was RM2,000. Since Ramesh has already met his deductible with the first treatment, the insurance company will cover a portion of the expenses. Ramesh will be responsible for 20% of the remaining RM500 (RM2,000 - RM1,500), which amounts to RM100. The insurance company will cover the remaining 80% of RM500, which amounts to RM400.

4. For the third treatment, the medical cost was RM1,700. Again, since Ramesh has already met his deductible, the insurance company will cover a portion of the expenses. Ramesh will be responsible for 20% of the entire cost, which amounts to RM340 (20% of RM1,700). The insurance company will cover the remaining 80% of RM1,700, which amounts to RM1,360.

5. To calculate the total medical expenses borne by Ramesh, we add up the amounts he is responsible for in each treatment: RM700 + RM100 + RM340 = RM1,140.

6. The total medical expenses covered by the insurance company can be calculated by adding up the amounts they pay in each treatment: RM400 + RM1,360 = RM1,760.

7. Finally, to find the overall amount borne by Ramesh and the insurance company, we sum up the individual expenses: Ramesh's expenses + insurance company's expenses = RM1,140 + RM1,760 = RM2,900.

For more such questions on expense, click on:

https://brainly.com/question/2292799

#SPJ8

carmen is taller than ramon but shorter than ricardo. carmen is shorter than ryan who is taller than ricardo. who is the tallest? who is the shortest?

Answers

Answer:

ramon is the shortest

ryan is the tallest

carmen is taller than ramon

carmen is shorter than ricardo

carmen is shorter than ryan

ryan is taller than ricardo

Solve the system of linear equations using substitution. Use a pencil and paper. Which expression would be easier to substitute into the other​ equation, in order to solve this​ problem? Explain your reasoning.
x=4y-9
x+4y=3

Answers

Answer:

(- 3, 1.5)

--------------------------

Given system:

x = 4y - 9x + 4y = 3

The first expression is ready to be substituted as no further operation is required to simplify it.

4y - 9 + 4y = 38y - 9 = 38y = 12y = 12/8y = 1.5

Find x:

x = 4*1.5 - 9x = 6 - 9x = - 3

The question is below

The question is below

Answers

Answer:

x + 7

Step-by-step explanation:

ajiahaqjjaqjajajanqjq

The ratio of the Cows and horses is 12:3 is there a 24 horses how many cows were would there be

Answers

Answer:

96

Step-by-step explanation:

Cows : Horses

12:3

x:24

(24/3) x 12

help please i need the answers in order please

A car was valued at $42,000 in the year 1994. The value depreciated to $11,000 by the year 2006.

A) What was the annual rate of change between 1994 and 2006?
r=------------ Round the rate of decrease to 4 decimal places.
B) What is the correct answer to part A written in percentage form?
r=------------%

C) Assume that the car value continues to drop by the same percentage. What will the value be in the year 2010 ?
value=$---------------- Round to the nearest 50 dollars.

Answers

A) To find the annual rate of change, we can use the formula:

r = (V2/V1)^(1/n) - 1

where:

V1 = initial value ($42,000)

V2 = final value ($11,000)

n = number of years (2006 - 1994 = 12)

Plugging in the values, we get:

r = (11000/42000)^(1/12) - 1 ≈ -0.1135

Therefore, the annual rate of change between 1994 and 2006 is approximately -0.1135.

B) To convert the rate of change to a percentage, we can multiply by 100 and add a percent sign:

r = -0.1135 × 100% ≈ -11.35%

Therefore, the correct answer to part A written in percentage form is approximately -11.35%.

C) Assuming the car value continues to drop by the same percentage, we can use the formula for exponential decay:

V = V0 * (1 - r)^t

where:

V0 = initial value ($11,000 in 2006)

r = annual rate of change (-0.1135)

t = number of years (2010 - 2006 = 4)

Plugging in the values, we get:

V = 11000 * (1 - (-0.1135))^4 ≈ $6,250

Therefore, the value of the car in the year 2010 would be approximately $6,250, rounded to the nearest 50 dollars.

PLS HELP ASAP I NEED IT PLS

PLS HELP ASAP I NEED IT PLS
PLS HELP ASAP I NEED IT PLS

Answers

Step-by-step explanation:

The equation for this function is y=2x

So 32*2=64

36/2=18

2*x=2x

y/2=y/2

2x*2=4x

(x+3)*2=2x+6


The perimeter of a rectangular garden is 26 ft. The length is 3 ft more than the width. Find the length

Answers

Answer:

Length = 8 ft.

Step-by-step explanation:

The perimeter of a rectangle can be represented by the formula P = 2w + 2l, where w is the width and l is the length.

Given what we're told, we can represent the length for this problem using the equation l = w + 3

Thus, to find the length, we plug in everything we know:

26 = 2(w +3) + 2w

26 = 2w + 6 + 2w

26 = 4w + 6

20 = 4w

5 = w

l = 5 + 3 = 8

Mr. Russo exchanged 200 euros (European money) for 270 dollars. How many dollars would
he get for 300 euros?

Answers

Answer: 405 dollars

Step-by-step explanation:

200 * x = 270

x = 270 * 300/200. x =405 dollars

The average value of a function f over the interval [−1,2] is −4, and the average value of f over the interval [2,7] is 8. What is the average value of f over the interval [−1,7] ?

Answers

The average value of a continuous function f(x) over an interval [a, b] is

\(\displaystyle f_{\mathrm{ave}[a,b]} = \frac1{b-a}\int_a^b f(x)\,dx\)

We're given that

\(\displaystyle f_{\rm ave[-1,2]} = \frac13 \int_{-1}^2 f(x) \, dx = -4\)

\(\displaystyle f_{\rm ave[2,7]} = \frac15 \int_2^7 f(x) \, dx = 8\)

and we want to determine

\(\displaystyle f_{\rm ave[-1,7]} = \frac18 \int_{-1}^7 f(x) \, dx\)

By the additive property of definite integration, we have

\(\displaystyle \int_{-1}^7 f(x) \, dx = \int_{-1}^2 f(x)\,dx + \int_2^7 f(x)\,dx\)

so it follows that

\(\displaystyle f_{\rm ave[-1,7]} = \frac18 \left(\int_{-1}^2 f(x)\,dx + \int_2^7 f(x)\,dx\right)\)

\(\displaystyle f_{\rm ave[-1,7]} = \frac18 \left(3\times(-4) + 5\times8\right)\)

\(\displaystyle f_{\rm ave[-1,7]} = \boxed{\frac72}\)

If 7 - 2x > 15, what is the solution for X?

Answers

Answer:

x<-4

Step-by-step explanation:

7 - 2x > 15

15+2x=7

x=-4

It has to be less than -4, so it can make the inequality true, you can plug in a number to the equations as well for example:

7-2(0)>15 Not True

7-2(-5)>15 True

PLZ HELP WILL GIVE BRAINLIEST!
What ratio do the squares have to the circles? Remember, it's SQUARES TO CIRCLES

PLZ HELP WILL GIVE BRAINLIEST!What ratio do the squares have to the circles? Remember, it's SQUARES TO

Answers

Answer:

A

Step-by-step explanation:

Answer:

this is Easy, the answer is A.

Step-by-step explanation:

just count the number of squares which is 3, and then count the number of circles which is 6.

on a separate sheet of paper, sketch the rectangle for each problem using any method round and estimate to check your answer problem 1 5 x 4,751

Answers

The rounded off answers for the area of the rectangles are as follows: 1) 24,000, 2) 42,000, 3) 31,200, 4) 31,200.

What is rounding of a number?

Rounding of a number is a mathematical process of approximating a given number to a specified level of accuracy or precision. Rounding is done to make numbers easier to work with or to communicate, especially when the number has many decimal places or digits.

The process of rounding involves changing a number to a nearby value that is easier to use or communicate, while still retaining its approximate value. The number is rounded to a certain number of decimal places or significant digits, depending on the required level of accuracy.

1. 5 x 4751 ≈ 5 x 4800 = 24,000.

To check the answer, we can estimate 4751 as 4800, and then multiply 5 by 4800 to get the approximate product of 24,000.

2. 7 x 6000 = 42,000.

To check the answer, we can simply multiply 7 by 6000 to get the product of 42,000.

3. 6 x 5214 ≈ 6 x 5200 = 31,200.

To check the answer, we can estimate 5214 as 5200, and then multiply 6 by 5200 to get the approximate product of 31,200.

4. 8 x 3867 ≈ 8 x 3900 = 31,200.

To check the answer, we can estimate 3867 as 3900, and then multiply 8 by 3900 to get the approximate product of 31,200.

To know more about significant digits, visit:

https://brainly.com/question/1658998

#SPJ1

The following are the scaled off results for the rectangles' area:: 1) 24,000, 2) 42,000, 3) 31,200, 4) 31,200.

What is rounding of a number?

The mathematical process of approximating a given number to a predetermined degree of accuracy or precision is known as rounding. In particular when a number has numerous decimal points or digits, rounding is done to make numbers simpler to work with or communicate.

In order to make a number simpler to use or communicate, a number is rounded to a more manageable value while retaining its general meaning.

Depending on the necessary level of accuracy, the number is rounded to a particular number of significant digits or decimal places.

1. 5 x 4751 ≈ 5 x 4800 = 24,000.

To verify the result, we can convert 4751 to 4800, then increase 5 by 4800 to obtain a result that is roughly 20,000.

2. 7 x 6000 = 42,000.

We can quickly multiply 7 by 6000 to obtain the result of 42,000 to verify the solution.

3. 6 x 5214 ≈ 6 x 5200 = 31,200.

By converting 5214 to 5200 and multiplying that number by 6, we can approximate the solution to be 31,200.

4. 8 x 3867 ≈ 8 x 3900 = 31,200.

To verify the solution, we can convert 3867 to 3900 and multiply 8 by 3900 to obtain a result that is roughly 31,200.

To know more about significant digits, visit:

brainly.com/question/1658998

#SPJ1

What does the y-intercept of the line tell you about the situation?
Please answer this quickly it’s due in 10min

What does the y-intercept of the line tell you about the situation?Please answer this quickly its due

Answers

The y-intercept of the linear function means that her initial distance from the finish line is of 10 kilometers.

How to define a linear function?

The slope-intercept equation for a linear function is presented as follows:

y = mx + b

In which:

m is the slope.b is the y-intercept.

The graph crosses the y-axis at y = 10, hence the intercept b is given as follows:

b = 10.

The y-values represent the distance in the context of this problem, hence the initial distance is of 10 km.

More can be learned about linear functions at https://brainly.com/question/15602982

#SPJ1

1. The demand of calculators when its price per unit is Rs 1200,is Rs 4000. When the price increases to Rs 1500, only 3000 calculators are demanded. a.Find the demand equation in the form of p = f(Q)
b.Obtain the number of calculations demanded when the price per unit calculators is Rs 1650
c. If 4500 calculations are demanded, what should be the price per unit of calculator? ​

Answers

a) the demand equation in the form of p = f(Q) is p = -10/3Q + 14533.33

b) when the price per unit of calculators is Rs 1650, approximately 3865 calculators are demanded.

c) when 4500 calculators are demanded, the price per unit should be approximately Rs 3010.

To find the demand equation in the form of p = f(Q), we need to determine the relationship between the price per unit (p) and the quantity demanded (Q) based on the given information.

Let's use the two data points provided:

Point 1: Price (p1) = Rs 1200, Quantity (Q1) = 4000

Point 2: Price (p2) = Rs 1500, Quantity (Q2) = 3000

First, we calculate the slope of the demand equation using the formula:

Slope = (Q2 - Q1) / (p2 - p1)

Substituting the values, we get:

Slope = (3000 - 4000) / (1500 - 1200) = -1000 / 300 = -10/3

Next, we use one of the data points and the slope to find the y-intercept (b). Let's use Point 1:

p1 = Rs 1200, Q1 = 4000

Using the equation of a straight line (y = mx + b), we can rearrange it to solve for b:

b = y - mx

b = 1200 - (-10/3)(4000) = 1200 + 40000/3 = 1200 + 13333.33 = 14533.33

Therefore, the demand equation in the form of p = f(Q) is:

p = -10/3Q + 14533.33

To find the number of calculators demanded (Q) when the price per unit is Rs 1650, we can rearrange the equation and solve for Q:

1650 = -10/3Q + 14533.33

-10/3Q = 1650 - 14533.33

-10/3Q = -12883.33

Q = (-12883.33) / (-10/3)

Q = 12883.33 * 3/10

Q ≈ 3865

Therefore, when the price per unit of calculators is Rs 1650, approximately 3865 calculators are demanded.

Now, let's determine the price per unit (p) when 4500 calculators are demanded. We rearrange the equation and solve for p:

4500 = -10/3Q + 14533.33

-10/3Q = 4500 - 14533.33

-10/3Q = -10033.33

Q = (-10033.33) / (-10/3)

Q = 10033.33 * 3/10

Q ≈ 3010

Therefore, when 4500 calculators are demanded, the price per unit should be approximately Rs 3010.

By using the slope-intercept form of a linear equation and the given data points, we can determine the demand equation and solve for various scenarios, including finding quantities demanded at specific prices and determining the appropriate price for a given quantity demanded.

Know more about quantity here:

https://brainly.com/question/13270106

#SPJ11

Divide the given polynomial by the given monomial.​

Divide the given polynomial by the given monomial.

Answers

Answer:

Here is the answer. Solved

Divide the given polynomial by the given monomial.

Which graph represents the function p(x) = |x – 1|?

On a coordinate plane, an absolute value graph has a vertex at (0, 1).

On a coordinate plane, an absolute value graph has a vertex at (negative 1, 0).

On a coordinate plane, an absolute value graph has a vertex at (0, negative 1).

Answers

The correct statement is: On a coordinate plane, an absolute value graph has a vertex at (0, 1).

The function p(x) = |x - 1| represents an absolute value function. The vertex of an absolute value function in the form f(x) = |x - h| + k is given by the point (h, k). In this case, the function p(x) = |x - 1| has a vertex at (1, 0).

Therefore, none of the provided options accurately represents the vertex of the function p(x) = |x - 1|. The correct vertex for this function is (1, 0), which means the vertex is at x = 1 and y = 0 on the coordinate plane. It is important to note that the vertex is located at (h, k) where h represents the x-coordinate and k represents the y-coordinate.

For more such questions on graph

https://brainly.com/question/19040584

#SPJ8

A $10,000 loan is to be repaid with 7 equal half-yearly instalments, the first repayment being in
6 months from today. Interest is at 7%p.a. compounding half-yearly. Calculate the principal
repaid in the fourth instalment.
(use excel)​

Answers

Answer:  $1,415.47

Step-by-step explanation:

see spreadsheet below

A $10,000 loan is to be repaid with 7 equal half-yearly instalments, the first repayment being in6 months

A wave travels 1/3 mile in 1/20 hour. At this rate, how far will the wave travel in 1 hour?​

Answers

Answer: 1/3*20=6.6666 or rounded 6.7mi. So the wave will travel 6.7mi in 1 hr or 6.6 repeating depending.

A wave will travel 6.7 mile in 1 hour.

Given that,

A wave travels 1/3 mile in 1/20 hour.

How to calculate the distance in given time?

To calculate the value of distance at time is given as,

Distance = Speed X Time

To given condition wave travels 1/3 mile in 1/20 hour,

We calculate,

⇒ \(\frac{1}{3}\) ÷ \(\frac{1}{20}\)

Divide a fraction by multiplying its reciprocal,

⇒ \(\frac{20}{3}\)

⇒ \(6.66667\)

⇒\(6.7\) mile

So a wave will travel 6.7 mile in 1 hour.

Learn more about the distance at a given time visit:

https://brainly.in/question/38241305

#SPJ2

What is a negative number minus a negative number?

Answers

Answer:

\(-x-(-y) = y-x\)

Step-by-step explanation:

lets say you have two negative numbers, -x and -y (x and y are positive).

\(-x-(-y) = y-x\)

Your answer is a positive number.

What angle is this need help asap

What angle is this need help asap

Answers

it’s an acute angle, but is almost a right angle. it isn’t a right angle because right angles are 90° , and usually have a box to show it.

Answer:

This is an acute angle, I accidently said it was a right angle before

What is cos(tan^-1(-2/3))=

Answers

cos(tan^(-1)(-2/3)) simplifies to 3√13 / 13.

To evaluate the expression cos(tan^(-1)(-2/3)), we can use the trigonometric identity:

cos(tan^(-1)(x)) = 1 / √(1 + x^2)

In this case, x is -2/3. Substituting the value into the identity:

cos(tan^(-1)(-2/3)) = 1 / √(1 + (-2/3)^2)

Now, let's calculate the value:

cos(tan^(-1)(-2/3)) = 1 / √(1 + 4/9)

                   = 1 / √(13/9)

                   = 1 / (√13/3)

                   = 3 / √13

                   = 3√13 / 13

For more such quaetions on simplifies

https://brainly.com/question/29369267

#SPJ8

Which of the following sets of interior angle measures can form a triangle?
Select all that apply.
A. 45, 45, 45*
B. 60, 60, 60°
C. 40.50,90°
D. 30.60.90*
E. 90,90°, 45°
F. 90, 90, 90°

pleaseee helppp

Which of the following sets of interior angle measures can form a triangle?Select all that apply.A. 45,

Answers

As per the angle sum property, the interior angles that form the triangle is option

B. 60, 60, 60°

C. 40.50,90°

D. 30.60.90*

Angle sum property

Angle sum property states that the sum of all the interior angles of the triangle is equal to 180 degrees.

Given,

Here we have the list of interior angles

A. 45, 45, 45*

B. 60, 60, 60°

C. 40.50,90°

D. 30.60.90*

E. 90,90°, 45°

F. 90, 90, 90°

Now, we have to find in which for these angle will form the tringle.

As per the definition of the angle sum property, the sum of all the interior angle of the triangle is 180°.

Based on these we have to verify each of the given interior angles,

A) 45 + 45 + 45 = 135

B. 60 + 60 + 60 = 180

C. 40 + 50 + 90 = 180

D. 30 + 60 + 90 = 180

E. 90 + 90 + 45 = 225

F. 90 + 90 + 90 = 270

Therefore, based on these values we have identified that the options (B), (C), and (D) forms the triangle.

To know more about Angle sum property here.

https://brainly.com/question/4316040

#SPJ1

Other Questions
1. A ride in a cab costs $0.60 plus $0.14 per mile. a. Write an equation for traveling x miles in the cab.b. The cab charges $0.88 for a ride of how many miles? c. How much does the cab charge for a trip of 8 miles? Consider the following series. k=0[infinity] xk/4k(k+7) (a) Use the Ratio Test to find the radius of convergence of the power series. Use the Ratio Test to find the interval of convergence of the power series. (Enter your answer using interval notation.) x (b) Use the Root Test to find the radius of convergence of the power series. Use the Root Test to find the interval of convergence of the power series. (Enter your answer using interval notation,) (c) Which test, the Ratio Test or the Root Test, did you find easier to use? Give the reasons why. please help due in 1 minute!! will give brainliest. 15 yo M presents with a one-year historyof failing grades, school absenteeism, andlegal problems, including shoplifting. His parents report that he spends most of his time alone in his room, adding that when he does go out, it is with a new set of friends. What is the most likely diagnosis? What is the most likely diagnosis? a rectangular tank that is 864 ft^3with a square base and open top is to be constructed of sheet steel of a given thickness. find the dimensions of the tank with minimum weight. If the volume of a rectangular prism is 1260 cubic feet and the length of the prism is 10 feet. What would be the measurement of thearea of the base? what's the purpose of thermometer in fractional distillation What is 2+2=4+6I will give brainlest Durante mi viaje a Chile, yo ________(tomar) un avin desde Santiago hasta la Isla de Pascua para ver las estatuas de Moai. Los Rapa Nui fueron una civilizacin antigua que construy estas estatuas maravillosas. Yo ________(tener) la oportunidad de tomar muchas fotos y las ________(compartir) con todos mis amigos. (3 points) Find the missing term represented by n:2(7 + 9) = 2(7) + 2 (n). 2 7 9 18 NO LINKS!!!Static electricity is a ____ charge.A. permanentB. missingC. flowingD. temporary Select the true statement about trend lines.O A. A trend line should be as close as possible to the points.O B. A trend line connects the points.O C. A trend line goes through the first and last points.D. A trend line is always horizontal.SUBMIT Select the best answer for the question4. Why were Africans enslaved and brought to the Americas?A. The Europeans struggled to enslave Native Americans because they were susceptible to disease and resisted enslavement.B. European monarchs were attempting to destroy all African societies so Europeans could live there.C. The Europeans admired the technological innovations of African societies.D. Europeans did not know how to function in the Americas and needed igenous groups for help HELP: If you multiply both sides of an inequality by a negative number when should you reverse the inequality symbol?A. SometimesB. AlwaysC. NeverD. Depends on if the number is a fraction of not Insurance can help with ??? An airplane has a mass of 12,000kg. If it is accelerating at 16m/sec2, what is its force?(round to the nearest whole number) NEED THIS ASAP please help i will give 30 points A popular fast-food restaurant serves cheeseburgers and fries. A family meal consistsof3 cheeseburgers and 3 orders of fries for a total of 4350 calories.A deluxe familymeal consists of 7 cheeseburgers and 5 orders of fries for a total of 9150 calories.How many calories arein one cheeseburger? Suppose that political instability in other countries makes people fear for the value of their assets in these countries so that they desire to purchase more U.S assets. What would the change in the exchange rate make happen to U.S. net exports and U.S. aggregate demand f (x)=2x-1find (-1)